How would someone destroy a black hole that’s at the centre of a planet? [duplicate]How would one destroy a black hole?How would one weaponize a black hole?Black hole launcherProtect a planet against a black hole attackHow would one destroy a black hole?Destroy a Planet inside a black hole?Black hole in ice-cream truck?If a planet couldn't escape a black hole, how would it survive going through one?Can a smaller black hole orbit a larger black hole?Building a planet with a primordial black hole coreBlack hole stardrives/artificial gravity

Is it possible to access the complete command line including pipes in a bash script?

How to Sow[] until I've Reap[]'d enough?

Company requiring me to let them review research from before I was hired

Why is the UH-60 tail rotor canted?

Killing a star safely

Is it ethical to tell my teaching assistant that I like him?

Would using carbon dioxide as fuel work to reduce the greenhouse effect?

If hash functions append the length, why does length extension attack work?

How can Kazakhstan perform MITM attacks on all HTTPS traffic?

How can I calculate the cost of Skyss bus tickets

Are stackless C++20 coroutines a problem?

Dedicated to our #1 Fan

Are there any English words pronounced with sounds/syllables that aren't part of the spelling?

Are rockets faster than airplanes?

Extrapolation v. Interpolation

Inverse Colombian Function

Can you find Airpod Case using Find my iPhone?

Strange LED behavior: Why is there a voltage over the LED with only one wire connected to it?

How did pilots avoid thunderstorms and related weather before “reliable” airborne weather radar was introduced on airliners?

I have a domain, static IP address and many devices I'd like to access outside my house. How do I route them?

Can't understand how static works exactly

Does quantity of data extensions impact performance?

What is "It is x o'clock" in Japanese with subject

Considerations when providing money to one child now, and the other later?



How would someone destroy a black hole that’s at the centre of a planet? [duplicate]


How would one destroy a black hole?How would one weaponize a black hole?Black hole launcherProtect a planet against a black hole attackHow would one destroy a black hole?Destroy a Planet inside a black hole?Black hole in ice-cream truck?If a planet couldn't escape a black hole, how would it survive going through one?Can a smaller black hole orbit a larger black hole?Building a planet with a primordial black hole coreBlack hole stardrives/artificial gravity






.everyoneloves__top-leaderboard:empty,.everyoneloves__mid-leaderboard:empty,.everyoneloves__bot-mid-leaderboard:empty margin-bottom:0;








13












$begingroup$



This question already has an answer here:



  • How would one destroy a black hole?

    13 answers



For starters, the black hole is temporarily contained. But once it’s containment fails it will devour the planet. Obviously, that’s a bad thing. What’s a reasonable way to prevent that?



Also the black hole is currently really tiny. Like 4 feet in diameter.



It can be an outlandish solution as long as it would have a chance to work, and would not take long to implement



Thanks.










share|improve this question









$endgroup$



marked as duplicate by Nzall, sphennings, Ash, EDL, Draco18s Jul 15 at 17:39


This question has been asked before and already has an answer. If those answers do not fully address your question, please ask a new question.













  • 2




    $begingroup$
    How did a black hole get in the center of a planet in the first place ? How it got there could relate directly to how to get it out of there. What's the planet made of - it's internal structure ? What tech level exists - e.g. could they "simply" dig a hole five feet wide to the center of a solid code and keep the tunnel from collapsing long enough to move the black hole ? The mass is also too large as pointed out others.
    $endgroup$
    – StephenG
    Jul 14 at 1:11










  • $begingroup$
    This question reminds me of that Stargate episode. And I wonder if it would have the same effect with time that the show had?
    $endgroup$
    – Trevor D
    Jul 14 at 1:54






  • 10




    $begingroup$
    A 4 feet black hole is in no way tiny. It's humongous!
    $endgroup$
    – vsz
    Jul 15 at 4:16






  • 4




    $begingroup$
    @R I did not mention the internal structure of the black hole, I mentioned the internal structure of the planet.
    $endgroup$
    – StephenG
    Jul 15 at 7:39






  • 5




    $begingroup$
    Possible duplicate of How would one destroy a black hole? I asked this question already a while ago. Could you indicate why that question does not provide you with any answers?
    $endgroup$
    – Nzall
    Jul 15 at 14:41


















13












$begingroup$



This question already has an answer here:



  • How would one destroy a black hole?

    13 answers



For starters, the black hole is temporarily contained. But once it’s containment fails it will devour the planet. Obviously, that’s a bad thing. What’s a reasonable way to prevent that?



Also the black hole is currently really tiny. Like 4 feet in diameter.



It can be an outlandish solution as long as it would have a chance to work, and would not take long to implement



Thanks.










share|improve this question









$endgroup$



marked as duplicate by Nzall, sphennings, Ash, EDL, Draco18s Jul 15 at 17:39


This question has been asked before and already has an answer. If those answers do not fully address your question, please ask a new question.













  • 2




    $begingroup$
    How did a black hole get in the center of a planet in the first place ? How it got there could relate directly to how to get it out of there. What's the planet made of - it's internal structure ? What tech level exists - e.g. could they "simply" dig a hole five feet wide to the center of a solid code and keep the tunnel from collapsing long enough to move the black hole ? The mass is also too large as pointed out others.
    $endgroup$
    – StephenG
    Jul 14 at 1:11










  • $begingroup$
    This question reminds me of that Stargate episode. And I wonder if it would have the same effect with time that the show had?
    $endgroup$
    – Trevor D
    Jul 14 at 1:54






  • 10




    $begingroup$
    A 4 feet black hole is in no way tiny. It's humongous!
    $endgroup$
    – vsz
    Jul 15 at 4:16






  • 4




    $begingroup$
    @R I did not mention the internal structure of the black hole, I mentioned the internal structure of the planet.
    $endgroup$
    – StephenG
    Jul 15 at 7:39






  • 5




    $begingroup$
    Possible duplicate of How would one destroy a black hole? I asked this question already a while ago. Could you indicate why that question does not provide you with any answers?
    $endgroup$
    – Nzall
    Jul 15 at 14:41














13












13








13


2



$begingroup$



This question already has an answer here:



  • How would one destroy a black hole?

    13 answers



For starters, the black hole is temporarily contained. But once it’s containment fails it will devour the planet. Obviously, that’s a bad thing. What’s a reasonable way to prevent that?



Also the black hole is currently really tiny. Like 4 feet in diameter.



It can be an outlandish solution as long as it would have a chance to work, and would not take long to implement



Thanks.










share|improve this question









$endgroup$





This question already has an answer here:



  • How would one destroy a black hole?

    13 answers



For starters, the black hole is temporarily contained. But once it’s containment fails it will devour the planet. Obviously, that’s a bad thing. What’s a reasonable way to prevent that?



Also the black hole is currently really tiny. Like 4 feet in diameter.



It can be an outlandish solution as long as it would have a chance to work, and would not take long to implement



Thanks.





This question already has an answer here:



  • How would one destroy a black hole?

    13 answers







planets science-fiction black-holes






share|improve this question













share|improve this question











share|improve this question




share|improve this question










asked Jul 14 at 0:26









Alisha SchneiderAlisha Schneider

661 silver badge4 bronze badges




661 silver badge4 bronze badges




marked as duplicate by Nzall, sphennings, Ash, EDL, Draco18s Jul 15 at 17:39


This question has been asked before and already has an answer. If those answers do not fully address your question, please ask a new question.









marked as duplicate by Nzall, sphennings, Ash, EDL, Draco18s Jul 15 at 17:39


This question has been asked before and already has an answer. If those answers do not fully address your question, please ask a new question.









  • 2




    $begingroup$
    How did a black hole get in the center of a planet in the first place ? How it got there could relate directly to how to get it out of there. What's the planet made of - it's internal structure ? What tech level exists - e.g. could they "simply" dig a hole five feet wide to the center of a solid code and keep the tunnel from collapsing long enough to move the black hole ? The mass is also too large as pointed out others.
    $endgroup$
    – StephenG
    Jul 14 at 1:11










  • $begingroup$
    This question reminds me of that Stargate episode. And I wonder if it would have the same effect with time that the show had?
    $endgroup$
    – Trevor D
    Jul 14 at 1:54






  • 10




    $begingroup$
    A 4 feet black hole is in no way tiny. It's humongous!
    $endgroup$
    – vsz
    Jul 15 at 4:16






  • 4




    $begingroup$
    @R I did not mention the internal structure of the black hole, I mentioned the internal structure of the planet.
    $endgroup$
    – StephenG
    Jul 15 at 7:39






  • 5




    $begingroup$
    Possible duplicate of How would one destroy a black hole? I asked this question already a while ago. Could you indicate why that question does not provide you with any answers?
    $endgroup$
    – Nzall
    Jul 15 at 14:41













  • 2




    $begingroup$
    How did a black hole get in the center of a planet in the first place ? How it got there could relate directly to how to get it out of there. What's the planet made of - it's internal structure ? What tech level exists - e.g. could they "simply" dig a hole five feet wide to the center of a solid code and keep the tunnel from collapsing long enough to move the black hole ? The mass is also too large as pointed out others.
    $endgroup$
    – StephenG
    Jul 14 at 1:11










  • $begingroup$
    This question reminds me of that Stargate episode. And I wonder if it would have the same effect with time that the show had?
    $endgroup$
    – Trevor D
    Jul 14 at 1:54






  • 10




    $begingroup$
    A 4 feet black hole is in no way tiny. It's humongous!
    $endgroup$
    – vsz
    Jul 15 at 4:16






  • 4




    $begingroup$
    @R I did not mention the internal structure of the black hole, I mentioned the internal structure of the planet.
    $endgroup$
    – StephenG
    Jul 15 at 7:39






  • 5




    $begingroup$
    Possible duplicate of How would one destroy a black hole? I asked this question already a while ago. Could you indicate why that question does not provide you with any answers?
    $endgroup$
    – Nzall
    Jul 15 at 14:41








2




2




$begingroup$
How did a black hole get in the center of a planet in the first place ? How it got there could relate directly to how to get it out of there. What's the planet made of - it's internal structure ? What tech level exists - e.g. could they "simply" dig a hole five feet wide to the center of a solid code and keep the tunnel from collapsing long enough to move the black hole ? The mass is also too large as pointed out others.
$endgroup$
– StephenG
Jul 14 at 1:11




$begingroup$
How did a black hole get in the center of a planet in the first place ? How it got there could relate directly to how to get it out of there. What's the planet made of - it's internal structure ? What tech level exists - e.g. could they "simply" dig a hole five feet wide to the center of a solid code and keep the tunnel from collapsing long enough to move the black hole ? The mass is also too large as pointed out others.
$endgroup$
– StephenG
Jul 14 at 1:11












$begingroup$
This question reminds me of that Stargate episode. And I wonder if it would have the same effect with time that the show had?
$endgroup$
– Trevor D
Jul 14 at 1:54




$begingroup$
This question reminds me of that Stargate episode. And I wonder if it would have the same effect with time that the show had?
$endgroup$
– Trevor D
Jul 14 at 1:54




10




10




$begingroup$
A 4 feet black hole is in no way tiny. It's humongous!
$endgroup$
– vsz
Jul 15 at 4:16




$begingroup$
A 4 feet black hole is in no way tiny. It's humongous!
$endgroup$
– vsz
Jul 15 at 4:16




4




4




$begingroup$
@R I did not mention the internal structure of the black hole, I mentioned the internal structure of the planet.
$endgroup$
– StephenG
Jul 15 at 7:39




$begingroup$
@R I did not mention the internal structure of the black hole, I mentioned the internal structure of the planet.
$endgroup$
– StephenG
Jul 15 at 7:39




5




5




$begingroup$
Possible duplicate of How would one destroy a black hole? I asked this question already a while ago. Could you indicate why that question does not provide you with any answers?
$endgroup$
– Nzall
Jul 15 at 14:41





$begingroup$
Possible duplicate of How would one destroy a black hole? I asked this question already a while ago. Could you indicate why that question does not provide you with any answers?
$endgroup$
– Nzall
Jul 15 at 14:41











6 Answers
6






active

oldest

votes


















33












$begingroup$

A black hole with a Schwarzschild radius of 4 feet would have a mass of 137.5 x Earth and the gravity to go along with it. Such a black hole would instantly liquefy the earth as it shreds it into an accretion disk and shoots gamma rays straight through the earth's poles.



There would be approximately 0 seconds for a committee to consider the problem before being vaporized, thus no solution will be found.



For narrative purposes however, the author might wish to reconsider the mass of the black hole to one that is less instantly catastrophic using the following: calculator. For instance, a black hole with a mass of 72135 metric tons would have a radius of 0.0001071103 femtometers and a lifetime of approximately 1 year before it goes critical and destroys the planet.



Such a black hole would be too small to interact with any atom inside the earth, so it would float freely, ping-ponging back and forth without ever acreting any more mass. A literal ticking-time-bomb if you will as it passes in and out, in and out through the Earth.






share|improve this answer











$endgroup$








  • 2




    $begingroup$
    @Zeiss In my example it would release energy of 1.6e16 calories per second. Enough to heat 1.6e13 kg of water 1 degree per second. However the mass of the earth is 5.9e24 kg so it should resist heating up that much for a good long time.
    $endgroup$
    – Benjamin
    Jul 14 at 0:54







  • 17




    $begingroup$
    If the black hole evaporated completely due to Hawking radiation, I don't think the intense burst of radiation at the end of its life would be enough to blow up the planet--this abstract of a Steven Hawking paper says in the last 0.1 seconds it would emit energy of 10^30 ergs = 10^23 joules, equivalent to about a million 1 megaton nuclear bombs, which would be a huge amount on the surface but I'm not sure how much we'd feel it if it happened at the core. By comparison, I've seen an estimate that says it's take about 10^32 joules to blow up the Earth.
    $endgroup$
    – Hypnosifl
    Jul 14 at 3:04






  • 3




    $begingroup$
    Also, the question said 4 feet in diameter, not 4 feet in radius. A Schwarzschild radius of 2 feet would mean a mass of 4.105 * 10^23 metric tons, the Earth's mass is 5.97 * 10^24 kg or 5.97 * 10^21 metric tons, so the black hole would be about 68.8 times more massive than the Earth.
    $endgroup$
    – Hypnosifl
    Jul 14 at 5:31






  • 3




    $begingroup$
    @Hypnosifl I was typing up a reply to disagree with you, but after calculating you seem to be correct that it's not enough energy. So I asked the question here instead: physics.stackexchange.com/q/491536/232868
    $endgroup$
    – Benjamin
    Jul 14 at 6:47






  • 3




    $begingroup$
    @Hypnosifl Even that much mass would do truly terrible things to the planet.
    $endgroup$
    – Ash
    Jul 14 at 10:45


















7












$begingroup$

Feed the hole negative mass.



Probably you have some negative matter around to power your Alcubierre drive. Load that into your little black hole. You will need a fair bit to make a difference.



To my surprise, this is apparently a done thing in theoretical circles: adding negative mass to model the shrinking of a black hole via Hawking radiation.



https://physics.stackexchange.com/questions/153376/what-would-happen-if-a-negative-mass-crossed-the-event-horizon-of-a-black-hole




The situation of negative mass falling into a black hole does have one
important consequence though, in GR it's the only way for the event
horizon of a black hole to shrink rather than expand, and for this
reason a dynamical black hole metric (the Vaidya metric) with negative
mass falling into it is sometimes used when trying to model the
long-term behavior of a black hole that is "evaporating" due to
continually emitting Hawking radiation (since this is a quantum
effect, and general relativity is not fully compatible with quantum
mechanics, this evaporation should ultimately require a full theory of
quantum gravity to model it completely accurately, but it seems
reasonable to expect that the earlier stages of evaporation, before
the size of the black hole and the energy density approach the Planck
scale where quantum gravity effects are expected to become
significant, should have some close analogue in classical general
relativity).







share|improve this answer









$endgroup$








  • 3




    $begingroup$
    Bad idea. Really, really bad idea. If you feed a black hole with positive mass, you reduce its Hawking radiation, making it more stable. If you feed it negative mass, you increase its Hawking radiation, pushing it further on the exponential path that will make it explode with the force of many, many atomic bombs. And the black hole becomes impossible to feed long before it blows up. Trust me, the last thing you want to do is sit on a black hole that's too small to feed. It would be like sitting in a submarine that's slowly sinking into the Mariana Trench with no means to stop its sinking...
    $endgroup$
    – cmaster
    Jul 14 at 17:49






  • 4




    $begingroup$
    Why not just put it in a handwavium box which shields against gravity? That's no more outre than negative mass!
    $endgroup$
    – Mark Olson
    Jul 14 at 19:41






  • 1




    $begingroup$
    @cmaster - See the comment by WetSavannaAnimal on the OP, if the exploding black hole releases about the same amount of energy as the asteroid that killed the dinosaurs, it seems unlikely it would cause devastating effects since even that much energy on the surface didn't break up the crust or turn it molten or anything, the harmful effects were mainly from the soot and there's no reason to think an explosion 6371 km underground would kick up soot in the same way. You might get big quakes and tsunamis, but a civilization with the tech to make negative mass could probably find tech solutions.
    $endgroup$
    – Hypnosifl
    Jul 14 at 20:57










  • $begingroup$
    @Hypnosifl The OP's black hole is so big it'll immediately turn the planet into an accretion disk. The people on that planet won't stand a chance to even realize that they are dying. My comment assumed that the black hole was scaled down to manageable levels in the first place. With black holes it's either that they are so small that you constantly need to feed them to avoid blow up, or that they are so stupendously heavy that nothing must get near them to avoid being torn apart by their tidal forces. Pick your poison.
    $endgroup$
    – cmaster
    Jul 14 at 21:16











  • $begingroup$
    @cmaster - Yes, I wasn't talking about the OP's exact scenario, just responding to your comment that feeding a more-reasonably-sized black hole (say, one with a radius of 0.1 mm) negative mass to shrink it would be a bad idea. My point is that although this would cause the black hole to blow up, if it would only release about as much energy as the impact of the chicxulub asteroid (in fact a little less than the asteroid, but same order of magnitude), then given that it'd be at the center of the Earth rather than the surface I don't think the consequences at the surface would be too terrible.
    $endgroup$
    – Hypnosifl
    Jul 14 at 21:30


















4












$begingroup$

I will assume you want something vaguely scientifically plausible. (I will also assume when you said "small black hole" you really meant one that is small enough to be contained on the surface of a planet and that the 4' size was not what you really meant.)



To start with, you can't destroy black holes. You can -- with difficulty -- move them. You can feed them and make them more massive. You can charge them and change their spin. But that's about it. So pretty much all you can do is try to get it up out of the planet, and that requires as much thrust as it would to launch the same mass of ordinary material into orbit and a way to apply that thrust to the BH.



(It's true that BHs probably evaporate due to Hawking radiation (though it's never been observed), but if HR is real, evaporation takes a very long time.)



The only way moving the BH might be doable (which wouldn't necessarily destroy the planet as a side-effect) involves charging the BH and then using electromagnetic fields to accelerate it. Pretty hard to do!



That brings up the question of the BH's mass. If it was temporarily contained, then it must have had a large charge and been supported by EM fields. It's mass is its mass, and if it was a small BH -- say the same mass as a mountain -- the containment facility would need to be able to support that mass concentrated into a really, really tiny object. The Schwarzschild radius of a BH of 100,000 tons (which would be very difficult to support with any imaginable technology) would be about 10-19 meters which is roughly 1 millionth the diameter of a proton.



Something that small would find it very difficult to swallow anything, so while it would orbit inside the planet, it would grow only very, very slowly. It would probably come to a halt (due to dynamical friction) at the planet's core and just sit there growing very, very, very slowly. It would probably have no effect whatsoever on a thousand year timescale.



The bigger the BH, the more of an impact it would have, but the less likely it would be that it could ever have been contained at the planet's surface.






share|improve this answer











$endgroup$








  • 1




    $begingroup$
    I seriously doubt that it's possible to impart a meaningful charge on a black hole: Any charge you add to it will be carried away by the hawking radiation rather quickly. Yes, it may absorb many billions of electrons, but the OP's black hole is many times heavier than earth. I guess, by the time you have it charged enough to be able to move earth relative to it, it will be spitting out its charge much faster than you can add charge to it.
    $endgroup$
    – cmaster
    Jul 14 at 17:55










  • $begingroup$
    @cmaster - There's an equation for the rate a charged black holes loses charge via Hawking radiation in this paper on sci-hub.tw, see equation (7) on p. 1144 (there's also a simpler approximate equation (9) but it only works when the black hole mass is significantly larger than 1.7 * 10^5 times the sun's mass, so it won't work for small black holes). Someone could use it to figure out the rate you'd need to fire charge in so it'd be greater than the rate of charge loss.
    $endgroup$
    – Hypnosifl
    Jul 15 at 17:49











  • $begingroup$
    Unfortunately, I don't speak Russian, so that paper is out-of-reach for me. Nevertheless, for black holes of stellar weight and more, charging them is no problem (ignoring the amount of charge you need to allow electrostatic manipulation) as they have negligible hawking radiation. It's the small, hot black holes that you might stand a chance to move if they were charged, which you cannot charge because they radiate their charge away.
    $endgroup$
    – cmaster
    Jul 15 at 18:34










  • $begingroup$
    @cmaster - the paper is not in russian, the sci-hub site will usually have a captcha test where the explanatory text is in russian but you just have to type the word you see in the graphic and then hit the big button. Technically it's a pirate site but it's widely used by scientists. Anyway, if you haven't done a calculation with an equation, how do you know the BH will radiate its charge away too quickly to do useful things like using its charge to transport it a distance similar to the radius of the Earth?
    $endgroup$
    – Hypnosifl
    Jul 15 at 21:00










  • $begingroup$
    @Hypnosifl Ah, well the captcha seems to have got stuck in my uMatrix - I only saw some Russian text... Anyway, have you ever tried levitating something by charging it? Unless the thing you are levitating is very light-weight, there's no chance to be successful. Not because the electric force is too weak, but because it's so strong and there are strongly charged particles everywhere which move to neutralize it. Ok, you can put your black hole into vacuum, but once the Hawking radiation becomes hot enough to include electrons/positrons, any charge will skew the emission rates, and soon be gone.
    $endgroup$
    – cmaster
    Jul 15 at 21:34


















4












$begingroup$

For something that sounds plausible, but totally isn't



Fire charged particles at relativistic speeds on tangents to the event horizon. And just before they cross the event horizon, slow them down to below relativistic speeds with bursts from powerful magnetic and electrical fields.



As they near the black hole, their induced mass creates gravitation fields to compete with the internal mass of the black hole. By firing multiple particles simultaneously at antipodal targets tangential to the event horizon, the black hole can't move so its internal mass it stretched between the equal gravitational forces pulling it apart.



By this means, tiny bits of matter can be teased away from the grip of the black hole.



Keep it up until it is too small to hurt the planet.



Scientists save the world, again. Your welcome.






share|improve this answer











$endgroup$








  • 3




    $begingroup$
    Sorry, but this won't work. The event horizon is really a point of no return, with the only exception of hawking radiation. Also, there's no such thing as "its internal mass it [sic] stretched between the equal gravitational forces". A black hole really behaves like a single dot of mass from any outside point of view. It cannot be deformed. You can deform the event horizon, as happens during black hole mergers, but that is purely an effect of spacetime around the black hole. The black hole itself keeps behaving as a single dot of mass.
    $endgroup$
    – cmaster
    Jul 14 at 18:03






  • 3




    $begingroup$
    @cmaster, please note the science-fiction tag and the opening clause of my answer -- its only meant to sound good. Of course, it won't work.
    $endgroup$
    – EDL
    Jul 14 at 18:26






  • 1




    $begingroup$
    Ok, I may not be the best audience for soft scifi as I've got quite a bit of physics training. And indeed, you say it's not plausible. Unfortunately, my downvote is already locked, so I can't remove it unless you edit your answer a bit. Nevertheless, the "sounds plausible" definitely does not extend to physics students.
    $endgroup$
    – cmaster
    Jul 14 at 18:41










  • $begingroup$
    @cmaster Could we reverse its polarity?
    $endgroup$
    – Sentry
    Jul 15 at 12:16










  • $begingroup$
    @Sentry Sorry, didn't understand your comment. Reverse the polarity of what?
    $endgroup$
    – cmaster
    Jul 15 at 16:45


















3












$begingroup$

A black hole with an event horizon circumference of ~13 feet (circumference because diameter is meaningless in a region of space so deeply curved) would mass far more than the Earth. The surface gravity of an Earth-sized planet with such a body at its core would be well beyond survivability of any land animal less tough than a small spider or insect (though aquatic creatures would be okay until they tried to dive too deep -- say, more than about ten meters).



So, don't worry about destroying that four foot black hole. Everyone will be dead before it's known why the Earth is being swallowed -- and it'll never get near the center of the planet; it'll tear the whole planet up before passing without a noticeable change in trajectory.






share|improve this answer











$endgroup$








  • 2




    $begingroup$
    diameter/radius is not meaningless, you just have to select a coordinate system, usually using the radial coordinate of Schwarzschild coordinates...I explained the physical meaning of the Schwarzschild radical coordinate here, it's defined by measuring the area and then using the standard relation Area = 4 * pi * radius^2
    $endgroup$
    – Hypnosifl
    Jul 14 at 5:02


















1












$begingroup$

Spin it up until its angular momentum becomes sufficiently large that it becomes a naked, toroidal singularity, at which point the laws of physics metaphorically throw up their hands and shrug in confusion.






share|improve this answer









$endgroup$








  • 3




    $begingroup$
    Which will instantly annihilate earth in a giant burst of gamma radiation...
    $endgroup$
    – cmaster
    Jul 14 at 18:04



















6 Answers
6






active

oldest

votes








6 Answers
6






active

oldest

votes









active

oldest

votes






active

oldest

votes









33












$begingroup$

A black hole with a Schwarzschild radius of 4 feet would have a mass of 137.5 x Earth and the gravity to go along with it. Such a black hole would instantly liquefy the earth as it shreds it into an accretion disk and shoots gamma rays straight through the earth's poles.



There would be approximately 0 seconds for a committee to consider the problem before being vaporized, thus no solution will be found.



For narrative purposes however, the author might wish to reconsider the mass of the black hole to one that is less instantly catastrophic using the following: calculator. For instance, a black hole with a mass of 72135 metric tons would have a radius of 0.0001071103 femtometers and a lifetime of approximately 1 year before it goes critical and destroys the planet.



Such a black hole would be too small to interact with any atom inside the earth, so it would float freely, ping-ponging back and forth without ever acreting any more mass. A literal ticking-time-bomb if you will as it passes in and out, in and out through the Earth.






share|improve this answer











$endgroup$








  • 2




    $begingroup$
    @Zeiss In my example it would release energy of 1.6e16 calories per second. Enough to heat 1.6e13 kg of water 1 degree per second. However the mass of the earth is 5.9e24 kg so it should resist heating up that much for a good long time.
    $endgroup$
    – Benjamin
    Jul 14 at 0:54







  • 17




    $begingroup$
    If the black hole evaporated completely due to Hawking radiation, I don't think the intense burst of radiation at the end of its life would be enough to blow up the planet--this abstract of a Steven Hawking paper says in the last 0.1 seconds it would emit energy of 10^30 ergs = 10^23 joules, equivalent to about a million 1 megaton nuclear bombs, which would be a huge amount on the surface but I'm not sure how much we'd feel it if it happened at the core. By comparison, I've seen an estimate that says it's take about 10^32 joules to blow up the Earth.
    $endgroup$
    – Hypnosifl
    Jul 14 at 3:04






  • 3




    $begingroup$
    Also, the question said 4 feet in diameter, not 4 feet in radius. A Schwarzschild radius of 2 feet would mean a mass of 4.105 * 10^23 metric tons, the Earth's mass is 5.97 * 10^24 kg or 5.97 * 10^21 metric tons, so the black hole would be about 68.8 times more massive than the Earth.
    $endgroup$
    – Hypnosifl
    Jul 14 at 5:31






  • 3




    $begingroup$
    @Hypnosifl I was typing up a reply to disagree with you, but after calculating you seem to be correct that it's not enough energy. So I asked the question here instead: physics.stackexchange.com/q/491536/232868
    $endgroup$
    – Benjamin
    Jul 14 at 6:47






  • 3




    $begingroup$
    @Hypnosifl Even that much mass would do truly terrible things to the planet.
    $endgroup$
    – Ash
    Jul 14 at 10:45















33












$begingroup$

A black hole with a Schwarzschild radius of 4 feet would have a mass of 137.5 x Earth and the gravity to go along with it. Such a black hole would instantly liquefy the earth as it shreds it into an accretion disk and shoots gamma rays straight through the earth's poles.



There would be approximately 0 seconds for a committee to consider the problem before being vaporized, thus no solution will be found.



For narrative purposes however, the author might wish to reconsider the mass of the black hole to one that is less instantly catastrophic using the following: calculator. For instance, a black hole with a mass of 72135 metric tons would have a radius of 0.0001071103 femtometers and a lifetime of approximately 1 year before it goes critical and destroys the planet.



Such a black hole would be too small to interact with any atom inside the earth, so it would float freely, ping-ponging back and forth without ever acreting any more mass. A literal ticking-time-bomb if you will as it passes in and out, in and out through the Earth.






share|improve this answer











$endgroup$








  • 2




    $begingroup$
    @Zeiss In my example it would release energy of 1.6e16 calories per second. Enough to heat 1.6e13 kg of water 1 degree per second. However the mass of the earth is 5.9e24 kg so it should resist heating up that much for a good long time.
    $endgroup$
    – Benjamin
    Jul 14 at 0:54







  • 17




    $begingroup$
    If the black hole evaporated completely due to Hawking radiation, I don't think the intense burst of radiation at the end of its life would be enough to blow up the planet--this abstract of a Steven Hawking paper says in the last 0.1 seconds it would emit energy of 10^30 ergs = 10^23 joules, equivalent to about a million 1 megaton nuclear bombs, which would be a huge amount on the surface but I'm not sure how much we'd feel it if it happened at the core. By comparison, I've seen an estimate that says it's take about 10^32 joules to blow up the Earth.
    $endgroup$
    – Hypnosifl
    Jul 14 at 3:04






  • 3




    $begingroup$
    Also, the question said 4 feet in diameter, not 4 feet in radius. A Schwarzschild radius of 2 feet would mean a mass of 4.105 * 10^23 metric tons, the Earth's mass is 5.97 * 10^24 kg or 5.97 * 10^21 metric tons, so the black hole would be about 68.8 times more massive than the Earth.
    $endgroup$
    – Hypnosifl
    Jul 14 at 5:31






  • 3




    $begingroup$
    @Hypnosifl I was typing up a reply to disagree with you, but after calculating you seem to be correct that it's not enough energy. So I asked the question here instead: physics.stackexchange.com/q/491536/232868
    $endgroup$
    – Benjamin
    Jul 14 at 6:47






  • 3




    $begingroup$
    @Hypnosifl Even that much mass would do truly terrible things to the planet.
    $endgroup$
    – Ash
    Jul 14 at 10:45













33












33








33





$begingroup$

A black hole with a Schwarzschild radius of 4 feet would have a mass of 137.5 x Earth and the gravity to go along with it. Such a black hole would instantly liquefy the earth as it shreds it into an accretion disk and shoots gamma rays straight through the earth's poles.



There would be approximately 0 seconds for a committee to consider the problem before being vaporized, thus no solution will be found.



For narrative purposes however, the author might wish to reconsider the mass of the black hole to one that is less instantly catastrophic using the following: calculator. For instance, a black hole with a mass of 72135 metric tons would have a radius of 0.0001071103 femtometers and a lifetime of approximately 1 year before it goes critical and destroys the planet.



Such a black hole would be too small to interact with any atom inside the earth, so it would float freely, ping-ponging back and forth without ever acreting any more mass. A literal ticking-time-bomb if you will as it passes in and out, in and out through the Earth.






share|improve this answer











$endgroup$



A black hole with a Schwarzschild radius of 4 feet would have a mass of 137.5 x Earth and the gravity to go along with it. Such a black hole would instantly liquefy the earth as it shreds it into an accretion disk and shoots gamma rays straight through the earth's poles.



There would be approximately 0 seconds for a committee to consider the problem before being vaporized, thus no solution will be found.



For narrative purposes however, the author might wish to reconsider the mass of the black hole to one that is less instantly catastrophic using the following: calculator. For instance, a black hole with a mass of 72135 metric tons would have a radius of 0.0001071103 femtometers and a lifetime of approximately 1 year before it goes critical and destroys the planet.



Such a black hole would be too small to interact with any atom inside the earth, so it would float freely, ping-ponging back and forth without ever acreting any more mass. A literal ticking-time-bomb if you will as it passes in and out, in and out through the Earth.







share|improve this answer














share|improve this answer



share|improve this answer








edited Jul 14 at 16:53

























answered Jul 14 at 0:40









BenjaminBenjamin

7263 silver badges16 bronze badges




7263 silver badges16 bronze badges







  • 2




    $begingroup$
    @Zeiss In my example it would release energy of 1.6e16 calories per second. Enough to heat 1.6e13 kg of water 1 degree per second. However the mass of the earth is 5.9e24 kg so it should resist heating up that much for a good long time.
    $endgroup$
    – Benjamin
    Jul 14 at 0:54







  • 17




    $begingroup$
    If the black hole evaporated completely due to Hawking radiation, I don't think the intense burst of radiation at the end of its life would be enough to blow up the planet--this abstract of a Steven Hawking paper says in the last 0.1 seconds it would emit energy of 10^30 ergs = 10^23 joules, equivalent to about a million 1 megaton nuclear bombs, which would be a huge amount on the surface but I'm not sure how much we'd feel it if it happened at the core. By comparison, I've seen an estimate that says it's take about 10^32 joules to blow up the Earth.
    $endgroup$
    – Hypnosifl
    Jul 14 at 3:04






  • 3




    $begingroup$
    Also, the question said 4 feet in diameter, not 4 feet in radius. A Schwarzschild radius of 2 feet would mean a mass of 4.105 * 10^23 metric tons, the Earth's mass is 5.97 * 10^24 kg or 5.97 * 10^21 metric tons, so the black hole would be about 68.8 times more massive than the Earth.
    $endgroup$
    – Hypnosifl
    Jul 14 at 5:31






  • 3




    $begingroup$
    @Hypnosifl I was typing up a reply to disagree with you, but after calculating you seem to be correct that it's not enough energy. So I asked the question here instead: physics.stackexchange.com/q/491536/232868
    $endgroup$
    – Benjamin
    Jul 14 at 6:47






  • 3




    $begingroup$
    @Hypnosifl Even that much mass would do truly terrible things to the planet.
    $endgroup$
    – Ash
    Jul 14 at 10:45












  • 2




    $begingroup$
    @Zeiss In my example it would release energy of 1.6e16 calories per second. Enough to heat 1.6e13 kg of water 1 degree per second. However the mass of the earth is 5.9e24 kg so it should resist heating up that much for a good long time.
    $endgroup$
    – Benjamin
    Jul 14 at 0:54







  • 17




    $begingroup$
    If the black hole evaporated completely due to Hawking radiation, I don't think the intense burst of radiation at the end of its life would be enough to blow up the planet--this abstract of a Steven Hawking paper says in the last 0.1 seconds it would emit energy of 10^30 ergs = 10^23 joules, equivalent to about a million 1 megaton nuclear bombs, which would be a huge amount on the surface but I'm not sure how much we'd feel it if it happened at the core. By comparison, I've seen an estimate that says it's take about 10^32 joules to blow up the Earth.
    $endgroup$
    – Hypnosifl
    Jul 14 at 3:04






  • 3




    $begingroup$
    Also, the question said 4 feet in diameter, not 4 feet in radius. A Schwarzschild radius of 2 feet would mean a mass of 4.105 * 10^23 metric tons, the Earth's mass is 5.97 * 10^24 kg or 5.97 * 10^21 metric tons, so the black hole would be about 68.8 times more massive than the Earth.
    $endgroup$
    – Hypnosifl
    Jul 14 at 5:31






  • 3




    $begingroup$
    @Hypnosifl I was typing up a reply to disagree with you, but after calculating you seem to be correct that it's not enough energy. So I asked the question here instead: physics.stackexchange.com/q/491536/232868
    $endgroup$
    – Benjamin
    Jul 14 at 6:47






  • 3




    $begingroup$
    @Hypnosifl Even that much mass would do truly terrible things to the planet.
    $endgroup$
    – Ash
    Jul 14 at 10:45







2




2




$begingroup$
@Zeiss In my example it would release energy of 1.6e16 calories per second. Enough to heat 1.6e13 kg of water 1 degree per second. However the mass of the earth is 5.9e24 kg so it should resist heating up that much for a good long time.
$endgroup$
– Benjamin
Jul 14 at 0:54





$begingroup$
@Zeiss In my example it would release energy of 1.6e16 calories per second. Enough to heat 1.6e13 kg of water 1 degree per second. However the mass of the earth is 5.9e24 kg so it should resist heating up that much for a good long time.
$endgroup$
– Benjamin
Jul 14 at 0:54





17




17




$begingroup$
If the black hole evaporated completely due to Hawking radiation, I don't think the intense burst of radiation at the end of its life would be enough to blow up the planet--this abstract of a Steven Hawking paper says in the last 0.1 seconds it would emit energy of 10^30 ergs = 10^23 joules, equivalent to about a million 1 megaton nuclear bombs, which would be a huge amount on the surface but I'm not sure how much we'd feel it if it happened at the core. By comparison, I've seen an estimate that says it's take about 10^32 joules to blow up the Earth.
$endgroup$
– Hypnosifl
Jul 14 at 3:04




$begingroup$
If the black hole evaporated completely due to Hawking radiation, I don't think the intense burst of radiation at the end of its life would be enough to blow up the planet--this abstract of a Steven Hawking paper says in the last 0.1 seconds it would emit energy of 10^30 ergs = 10^23 joules, equivalent to about a million 1 megaton nuclear bombs, which would be a huge amount on the surface but I'm not sure how much we'd feel it if it happened at the core. By comparison, I've seen an estimate that says it's take about 10^32 joules to blow up the Earth.
$endgroup$
– Hypnosifl
Jul 14 at 3:04




3




3




$begingroup$
Also, the question said 4 feet in diameter, not 4 feet in radius. A Schwarzschild radius of 2 feet would mean a mass of 4.105 * 10^23 metric tons, the Earth's mass is 5.97 * 10^24 kg or 5.97 * 10^21 metric tons, so the black hole would be about 68.8 times more massive than the Earth.
$endgroup$
– Hypnosifl
Jul 14 at 5:31




$begingroup$
Also, the question said 4 feet in diameter, not 4 feet in radius. A Schwarzschild radius of 2 feet would mean a mass of 4.105 * 10^23 metric tons, the Earth's mass is 5.97 * 10^24 kg or 5.97 * 10^21 metric tons, so the black hole would be about 68.8 times more massive than the Earth.
$endgroup$
– Hypnosifl
Jul 14 at 5:31




3




3




$begingroup$
@Hypnosifl I was typing up a reply to disagree with you, but after calculating you seem to be correct that it's not enough energy. So I asked the question here instead: physics.stackexchange.com/q/491536/232868
$endgroup$
– Benjamin
Jul 14 at 6:47




$begingroup$
@Hypnosifl I was typing up a reply to disagree with you, but after calculating you seem to be correct that it's not enough energy. So I asked the question here instead: physics.stackexchange.com/q/491536/232868
$endgroup$
– Benjamin
Jul 14 at 6:47




3




3




$begingroup$
@Hypnosifl Even that much mass would do truly terrible things to the planet.
$endgroup$
– Ash
Jul 14 at 10:45




$begingroup$
@Hypnosifl Even that much mass would do truly terrible things to the planet.
$endgroup$
– Ash
Jul 14 at 10:45













7












$begingroup$

Feed the hole negative mass.



Probably you have some negative matter around to power your Alcubierre drive. Load that into your little black hole. You will need a fair bit to make a difference.



To my surprise, this is apparently a done thing in theoretical circles: adding negative mass to model the shrinking of a black hole via Hawking radiation.



https://physics.stackexchange.com/questions/153376/what-would-happen-if-a-negative-mass-crossed-the-event-horizon-of-a-black-hole




The situation of negative mass falling into a black hole does have one
important consequence though, in GR it's the only way for the event
horizon of a black hole to shrink rather than expand, and for this
reason a dynamical black hole metric (the Vaidya metric) with negative
mass falling into it is sometimes used when trying to model the
long-term behavior of a black hole that is "evaporating" due to
continually emitting Hawking radiation (since this is a quantum
effect, and general relativity is not fully compatible with quantum
mechanics, this evaporation should ultimately require a full theory of
quantum gravity to model it completely accurately, but it seems
reasonable to expect that the earlier stages of evaporation, before
the size of the black hole and the energy density approach the Planck
scale where quantum gravity effects are expected to become
significant, should have some close analogue in classical general
relativity).







share|improve this answer









$endgroup$








  • 3




    $begingroup$
    Bad idea. Really, really bad idea. If you feed a black hole with positive mass, you reduce its Hawking radiation, making it more stable. If you feed it negative mass, you increase its Hawking radiation, pushing it further on the exponential path that will make it explode with the force of many, many atomic bombs. And the black hole becomes impossible to feed long before it blows up. Trust me, the last thing you want to do is sit on a black hole that's too small to feed. It would be like sitting in a submarine that's slowly sinking into the Mariana Trench with no means to stop its sinking...
    $endgroup$
    – cmaster
    Jul 14 at 17:49






  • 4




    $begingroup$
    Why not just put it in a handwavium box which shields against gravity? That's no more outre than negative mass!
    $endgroup$
    – Mark Olson
    Jul 14 at 19:41






  • 1




    $begingroup$
    @cmaster - See the comment by WetSavannaAnimal on the OP, if the exploding black hole releases about the same amount of energy as the asteroid that killed the dinosaurs, it seems unlikely it would cause devastating effects since even that much energy on the surface didn't break up the crust or turn it molten or anything, the harmful effects were mainly from the soot and there's no reason to think an explosion 6371 km underground would kick up soot in the same way. You might get big quakes and tsunamis, but a civilization with the tech to make negative mass could probably find tech solutions.
    $endgroup$
    – Hypnosifl
    Jul 14 at 20:57










  • $begingroup$
    @Hypnosifl The OP's black hole is so big it'll immediately turn the planet into an accretion disk. The people on that planet won't stand a chance to even realize that they are dying. My comment assumed that the black hole was scaled down to manageable levels in the first place. With black holes it's either that they are so small that you constantly need to feed them to avoid blow up, or that they are so stupendously heavy that nothing must get near them to avoid being torn apart by their tidal forces. Pick your poison.
    $endgroup$
    – cmaster
    Jul 14 at 21:16











  • $begingroup$
    @cmaster - Yes, I wasn't talking about the OP's exact scenario, just responding to your comment that feeding a more-reasonably-sized black hole (say, one with a radius of 0.1 mm) negative mass to shrink it would be a bad idea. My point is that although this would cause the black hole to blow up, if it would only release about as much energy as the impact of the chicxulub asteroid (in fact a little less than the asteroid, but same order of magnitude), then given that it'd be at the center of the Earth rather than the surface I don't think the consequences at the surface would be too terrible.
    $endgroup$
    – Hypnosifl
    Jul 14 at 21:30















7












$begingroup$

Feed the hole negative mass.



Probably you have some negative matter around to power your Alcubierre drive. Load that into your little black hole. You will need a fair bit to make a difference.



To my surprise, this is apparently a done thing in theoretical circles: adding negative mass to model the shrinking of a black hole via Hawking radiation.



https://physics.stackexchange.com/questions/153376/what-would-happen-if-a-negative-mass-crossed-the-event-horizon-of-a-black-hole




The situation of negative mass falling into a black hole does have one
important consequence though, in GR it's the only way for the event
horizon of a black hole to shrink rather than expand, and for this
reason a dynamical black hole metric (the Vaidya metric) with negative
mass falling into it is sometimes used when trying to model the
long-term behavior of a black hole that is "evaporating" due to
continually emitting Hawking radiation (since this is a quantum
effect, and general relativity is not fully compatible with quantum
mechanics, this evaporation should ultimately require a full theory of
quantum gravity to model it completely accurately, but it seems
reasonable to expect that the earlier stages of evaporation, before
the size of the black hole and the energy density approach the Planck
scale where quantum gravity effects are expected to become
significant, should have some close analogue in classical general
relativity).







share|improve this answer









$endgroup$








  • 3




    $begingroup$
    Bad idea. Really, really bad idea. If you feed a black hole with positive mass, you reduce its Hawking radiation, making it more stable. If you feed it negative mass, you increase its Hawking radiation, pushing it further on the exponential path that will make it explode with the force of many, many atomic bombs. And the black hole becomes impossible to feed long before it blows up. Trust me, the last thing you want to do is sit on a black hole that's too small to feed. It would be like sitting in a submarine that's slowly sinking into the Mariana Trench with no means to stop its sinking...
    $endgroup$
    – cmaster
    Jul 14 at 17:49






  • 4




    $begingroup$
    Why not just put it in a handwavium box which shields against gravity? That's no more outre than negative mass!
    $endgroup$
    – Mark Olson
    Jul 14 at 19:41






  • 1




    $begingroup$
    @cmaster - See the comment by WetSavannaAnimal on the OP, if the exploding black hole releases about the same amount of energy as the asteroid that killed the dinosaurs, it seems unlikely it would cause devastating effects since even that much energy on the surface didn't break up the crust or turn it molten or anything, the harmful effects were mainly from the soot and there's no reason to think an explosion 6371 km underground would kick up soot in the same way. You might get big quakes and tsunamis, but a civilization with the tech to make negative mass could probably find tech solutions.
    $endgroup$
    – Hypnosifl
    Jul 14 at 20:57










  • $begingroup$
    @Hypnosifl The OP's black hole is so big it'll immediately turn the planet into an accretion disk. The people on that planet won't stand a chance to even realize that they are dying. My comment assumed that the black hole was scaled down to manageable levels in the first place. With black holes it's either that they are so small that you constantly need to feed them to avoid blow up, or that they are so stupendously heavy that nothing must get near them to avoid being torn apart by their tidal forces. Pick your poison.
    $endgroup$
    – cmaster
    Jul 14 at 21:16











  • $begingroup$
    @cmaster - Yes, I wasn't talking about the OP's exact scenario, just responding to your comment that feeding a more-reasonably-sized black hole (say, one with a radius of 0.1 mm) negative mass to shrink it would be a bad idea. My point is that although this would cause the black hole to blow up, if it would only release about as much energy as the impact of the chicxulub asteroid (in fact a little less than the asteroid, but same order of magnitude), then given that it'd be at the center of the Earth rather than the surface I don't think the consequences at the surface would be too terrible.
    $endgroup$
    – Hypnosifl
    Jul 14 at 21:30













7












7








7





$begingroup$

Feed the hole negative mass.



Probably you have some negative matter around to power your Alcubierre drive. Load that into your little black hole. You will need a fair bit to make a difference.



To my surprise, this is apparently a done thing in theoretical circles: adding negative mass to model the shrinking of a black hole via Hawking radiation.



https://physics.stackexchange.com/questions/153376/what-would-happen-if-a-negative-mass-crossed-the-event-horizon-of-a-black-hole




The situation of negative mass falling into a black hole does have one
important consequence though, in GR it's the only way for the event
horizon of a black hole to shrink rather than expand, and for this
reason a dynamical black hole metric (the Vaidya metric) with negative
mass falling into it is sometimes used when trying to model the
long-term behavior of a black hole that is "evaporating" due to
continually emitting Hawking radiation (since this is a quantum
effect, and general relativity is not fully compatible with quantum
mechanics, this evaporation should ultimately require a full theory of
quantum gravity to model it completely accurately, but it seems
reasonable to expect that the earlier stages of evaporation, before
the size of the black hole and the energy density approach the Planck
scale where quantum gravity effects are expected to become
significant, should have some close analogue in classical general
relativity).







share|improve this answer









$endgroup$



Feed the hole negative mass.



Probably you have some negative matter around to power your Alcubierre drive. Load that into your little black hole. You will need a fair bit to make a difference.



To my surprise, this is apparently a done thing in theoretical circles: adding negative mass to model the shrinking of a black hole via Hawking radiation.



https://physics.stackexchange.com/questions/153376/what-would-happen-if-a-negative-mass-crossed-the-event-horizon-of-a-black-hole




The situation of negative mass falling into a black hole does have one
important consequence though, in GR it's the only way for the event
horizon of a black hole to shrink rather than expand, and for this
reason a dynamical black hole metric (the Vaidya metric) with negative
mass falling into it is sometimes used when trying to model the
long-term behavior of a black hole that is "evaporating" due to
continually emitting Hawking radiation (since this is a quantum
effect, and general relativity is not fully compatible with quantum
mechanics, this evaporation should ultimately require a full theory of
quantum gravity to model it completely accurately, but it seems
reasonable to expect that the earlier stages of evaporation, before
the size of the black hole and the energy density approach the Planck
scale where quantum gravity effects are expected to become
significant, should have some close analogue in classical general
relativity).








share|improve this answer












share|improve this answer



share|improve this answer










answered Jul 14 at 5:25









WillkWillk

131k33 gold badges246 silver badges546 bronze badges




131k33 gold badges246 silver badges546 bronze badges







  • 3




    $begingroup$
    Bad idea. Really, really bad idea. If you feed a black hole with positive mass, you reduce its Hawking radiation, making it more stable. If you feed it negative mass, you increase its Hawking radiation, pushing it further on the exponential path that will make it explode with the force of many, many atomic bombs. And the black hole becomes impossible to feed long before it blows up. Trust me, the last thing you want to do is sit on a black hole that's too small to feed. It would be like sitting in a submarine that's slowly sinking into the Mariana Trench with no means to stop its sinking...
    $endgroup$
    – cmaster
    Jul 14 at 17:49






  • 4




    $begingroup$
    Why not just put it in a handwavium box which shields against gravity? That's no more outre than negative mass!
    $endgroup$
    – Mark Olson
    Jul 14 at 19:41






  • 1




    $begingroup$
    @cmaster - See the comment by WetSavannaAnimal on the OP, if the exploding black hole releases about the same amount of energy as the asteroid that killed the dinosaurs, it seems unlikely it would cause devastating effects since even that much energy on the surface didn't break up the crust or turn it molten or anything, the harmful effects were mainly from the soot and there's no reason to think an explosion 6371 km underground would kick up soot in the same way. You might get big quakes and tsunamis, but a civilization with the tech to make negative mass could probably find tech solutions.
    $endgroup$
    – Hypnosifl
    Jul 14 at 20:57










  • $begingroup$
    @Hypnosifl The OP's black hole is so big it'll immediately turn the planet into an accretion disk. The people on that planet won't stand a chance to even realize that they are dying. My comment assumed that the black hole was scaled down to manageable levels in the first place. With black holes it's either that they are so small that you constantly need to feed them to avoid blow up, or that they are so stupendously heavy that nothing must get near them to avoid being torn apart by their tidal forces. Pick your poison.
    $endgroup$
    – cmaster
    Jul 14 at 21:16











  • $begingroup$
    @cmaster - Yes, I wasn't talking about the OP's exact scenario, just responding to your comment that feeding a more-reasonably-sized black hole (say, one with a radius of 0.1 mm) negative mass to shrink it would be a bad idea. My point is that although this would cause the black hole to blow up, if it would only release about as much energy as the impact of the chicxulub asteroid (in fact a little less than the asteroid, but same order of magnitude), then given that it'd be at the center of the Earth rather than the surface I don't think the consequences at the surface would be too terrible.
    $endgroup$
    – Hypnosifl
    Jul 14 at 21:30












  • 3




    $begingroup$
    Bad idea. Really, really bad idea. If you feed a black hole with positive mass, you reduce its Hawking radiation, making it more stable. If you feed it negative mass, you increase its Hawking radiation, pushing it further on the exponential path that will make it explode with the force of many, many atomic bombs. And the black hole becomes impossible to feed long before it blows up. Trust me, the last thing you want to do is sit on a black hole that's too small to feed. It would be like sitting in a submarine that's slowly sinking into the Mariana Trench with no means to stop its sinking...
    $endgroup$
    – cmaster
    Jul 14 at 17:49






  • 4




    $begingroup$
    Why not just put it in a handwavium box which shields against gravity? That's no more outre than negative mass!
    $endgroup$
    – Mark Olson
    Jul 14 at 19:41






  • 1




    $begingroup$
    @cmaster - See the comment by WetSavannaAnimal on the OP, if the exploding black hole releases about the same amount of energy as the asteroid that killed the dinosaurs, it seems unlikely it would cause devastating effects since even that much energy on the surface didn't break up the crust or turn it molten or anything, the harmful effects were mainly from the soot and there's no reason to think an explosion 6371 km underground would kick up soot in the same way. You might get big quakes and tsunamis, but a civilization with the tech to make negative mass could probably find tech solutions.
    $endgroup$
    – Hypnosifl
    Jul 14 at 20:57










  • $begingroup$
    @Hypnosifl The OP's black hole is so big it'll immediately turn the planet into an accretion disk. The people on that planet won't stand a chance to even realize that they are dying. My comment assumed that the black hole was scaled down to manageable levels in the first place. With black holes it's either that they are so small that you constantly need to feed them to avoid blow up, or that they are so stupendously heavy that nothing must get near them to avoid being torn apart by their tidal forces. Pick your poison.
    $endgroup$
    – cmaster
    Jul 14 at 21:16











  • $begingroup$
    @cmaster - Yes, I wasn't talking about the OP's exact scenario, just responding to your comment that feeding a more-reasonably-sized black hole (say, one with a radius of 0.1 mm) negative mass to shrink it would be a bad idea. My point is that although this would cause the black hole to blow up, if it would only release about as much energy as the impact of the chicxulub asteroid (in fact a little less than the asteroid, but same order of magnitude), then given that it'd be at the center of the Earth rather than the surface I don't think the consequences at the surface would be too terrible.
    $endgroup$
    – Hypnosifl
    Jul 14 at 21:30







3




3




$begingroup$
Bad idea. Really, really bad idea. If you feed a black hole with positive mass, you reduce its Hawking radiation, making it more stable. If you feed it negative mass, you increase its Hawking radiation, pushing it further on the exponential path that will make it explode with the force of many, many atomic bombs. And the black hole becomes impossible to feed long before it blows up. Trust me, the last thing you want to do is sit on a black hole that's too small to feed. It would be like sitting in a submarine that's slowly sinking into the Mariana Trench with no means to stop its sinking...
$endgroup$
– cmaster
Jul 14 at 17:49




$begingroup$
Bad idea. Really, really bad idea. If you feed a black hole with positive mass, you reduce its Hawking radiation, making it more stable. If you feed it negative mass, you increase its Hawking radiation, pushing it further on the exponential path that will make it explode with the force of many, many atomic bombs. And the black hole becomes impossible to feed long before it blows up. Trust me, the last thing you want to do is sit on a black hole that's too small to feed. It would be like sitting in a submarine that's slowly sinking into the Mariana Trench with no means to stop its sinking...
$endgroup$
– cmaster
Jul 14 at 17:49




4




4




$begingroup$
Why not just put it in a handwavium box which shields against gravity? That's no more outre than negative mass!
$endgroup$
– Mark Olson
Jul 14 at 19:41




$begingroup$
Why not just put it in a handwavium box which shields against gravity? That's no more outre than negative mass!
$endgroup$
– Mark Olson
Jul 14 at 19:41




1




1




$begingroup$
@cmaster - See the comment by WetSavannaAnimal on the OP, if the exploding black hole releases about the same amount of energy as the asteroid that killed the dinosaurs, it seems unlikely it would cause devastating effects since even that much energy on the surface didn't break up the crust or turn it molten or anything, the harmful effects were mainly from the soot and there's no reason to think an explosion 6371 km underground would kick up soot in the same way. You might get big quakes and tsunamis, but a civilization with the tech to make negative mass could probably find tech solutions.
$endgroup$
– Hypnosifl
Jul 14 at 20:57




$begingroup$
@cmaster - See the comment by WetSavannaAnimal on the OP, if the exploding black hole releases about the same amount of energy as the asteroid that killed the dinosaurs, it seems unlikely it would cause devastating effects since even that much energy on the surface didn't break up the crust or turn it molten or anything, the harmful effects were mainly from the soot and there's no reason to think an explosion 6371 km underground would kick up soot in the same way. You might get big quakes and tsunamis, but a civilization with the tech to make negative mass could probably find tech solutions.
$endgroup$
– Hypnosifl
Jul 14 at 20:57












$begingroup$
@Hypnosifl The OP's black hole is so big it'll immediately turn the planet into an accretion disk. The people on that planet won't stand a chance to even realize that they are dying. My comment assumed that the black hole was scaled down to manageable levels in the first place. With black holes it's either that they are so small that you constantly need to feed them to avoid blow up, or that they are so stupendously heavy that nothing must get near them to avoid being torn apart by their tidal forces. Pick your poison.
$endgroup$
– cmaster
Jul 14 at 21:16





$begingroup$
@Hypnosifl The OP's black hole is so big it'll immediately turn the planet into an accretion disk. The people on that planet won't stand a chance to even realize that they are dying. My comment assumed that the black hole was scaled down to manageable levels in the first place. With black holes it's either that they are so small that you constantly need to feed them to avoid blow up, or that they are so stupendously heavy that nothing must get near them to avoid being torn apart by their tidal forces. Pick your poison.
$endgroup$
– cmaster
Jul 14 at 21:16













$begingroup$
@cmaster - Yes, I wasn't talking about the OP's exact scenario, just responding to your comment that feeding a more-reasonably-sized black hole (say, one with a radius of 0.1 mm) negative mass to shrink it would be a bad idea. My point is that although this would cause the black hole to blow up, if it would only release about as much energy as the impact of the chicxulub asteroid (in fact a little less than the asteroid, but same order of magnitude), then given that it'd be at the center of the Earth rather than the surface I don't think the consequences at the surface would be too terrible.
$endgroup$
– Hypnosifl
Jul 14 at 21:30




$begingroup$
@cmaster - Yes, I wasn't talking about the OP's exact scenario, just responding to your comment that feeding a more-reasonably-sized black hole (say, one with a radius of 0.1 mm) negative mass to shrink it would be a bad idea. My point is that although this would cause the black hole to blow up, if it would only release about as much energy as the impact of the chicxulub asteroid (in fact a little less than the asteroid, but same order of magnitude), then given that it'd be at the center of the Earth rather than the surface I don't think the consequences at the surface would be too terrible.
$endgroup$
– Hypnosifl
Jul 14 at 21:30











4












$begingroup$

I will assume you want something vaguely scientifically plausible. (I will also assume when you said "small black hole" you really meant one that is small enough to be contained on the surface of a planet and that the 4' size was not what you really meant.)



To start with, you can't destroy black holes. You can -- with difficulty -- move them. You can feed them and make them more massive. You can charge them and change their spin. But that's about it. So pretty much all you can do is try to get it up out of the planet, and that requires as much thrust as it would to launch the same mass of ordinary material into orbit and a way to apply that thrust to the BH.



(It's true that BHs probably evaporate due to Hawking radiation (though it's never been observed), but if HR is real, evaporation takes a very long time.)



The only way moving the BH might be doable (which wouldn't necessarily destroy the planet as a side-effect) involves charging the BH and then using electromagnetic fields to accelerate it. Pretty hard to do!



That brings up the question of the BH's mass. If it was temporarily contained, then it must have had a large charge and been supported by EM fields. It's mass is its mass, and if it was a small BH -- say the same mass as a mountain -- the containment facility would need to be able to support that mass concentrated into a really, really tiny object. The Schwarzschild radius of a BH of 100,000 tons (which would be very difficult to support with any imaginable technology) would be about 10-19 meters which is roughly 1 millionth the diameter of a proton.



Something that small would find it very difficult to swallow anything, so while it would orbit inside the planet, it would grow only very, very slowly. It would probably come to a halt (due to dynamical friction) at the planet's core and just sit there growing very, very, very slowly. It would probably have no effect whatsoever on a thousand year timescale.



The bigger the BH, the more of an impact it would have, but the less likely it would be that it could ever have been contained at the planet's surface.






share|improve this answer











$endgroup$








  • 1




    $begingroup$
    I seriously doubt that it's possible to impart a meaningful charge on a black hole: Any charge you add to it will be carried away by the hawking radiation rather quickly. Yes, it may absorb many billions of electrons, but the OP's black hole is many times heavier than earth. I guess, by the time you have it charged enough to be able to move earth relative to it, it will be spitting out its charge much faster than you can add charge to it.
    $endgroup$
    – cmaster
    Jul 14 at 17:55










  • $begingroup$
    @cmaster - There's an equation for the rate a charged black holes loses charge via Hawking radiation in this paper on sci-hub.tw, see equation (7) on p. 1144 (there's also a simpler approximate equation (9) but it only works when the black hole mass is significantly larger than 1.7 * 10^5 times the sun's mass, so it won't work for small black holes). Someone could use it to figure out the rate you'd need to fire charge in so it'd be greater than the rate of charge loss.
    $endgroup$
    – Hypnosifl
    Jul 15 at 17:49











  • $begingroup$
    Unfortunately, I don't speak Russian, so that paper is out-of-reach for me. Nevertheless, for black holes of stellar weight and more, charging them is no problem (ignoring the amount of charge you need to allow electrostatic manipulation) as they have negligible hawking radiation. It's the small, hot black holes that you might stand a chance to move if they were charged, which you cannot charge because they radiate their charge away.
    $endgroup$
    – cmaster
    Jul 15 at 18:34










  • $begingroup$
    @cmaster - the paper is not in russian, the sci-hub site will usually have a captcha test where the explanatory text is in russian but you just have to type the word you see in the graphic and then hit the big button. Technically it's a pirate site but it's widely used by scientists. Anyway, if you haven't done a calculation with an equation, how do you know the BH will radiate its charge away too quickly to do useful things like using its charge to transport it a distance similar to the radius of the Earth?
    $endgroup$
    – Hypnosifl
    Jul 15 at 21:00










  • $begingroup$
    @Hypnosifl Ah, well the captcha seems to have got stuck in my uMatrix - I only saw some Russian text... Anyway, have you ever tried levitating something by charging it? Unless the thing you are levitating is very light-weight, there's no chance to be successful. Not because the electric force is too weak, but because it's so strong and there are strongly charged particles everywhere which move to neutralize it. Ok, you can put your black hole into vacuum, but once the Hawking radiation becomes hot enough to include electrons/positrons, any charge will skew the emission rates, and soon be gone.
    $endgroup$
    – cmaster
    Jul 15 at 21:34















4












$begingroup$

I will assume you want something vaguely scientifically plausible. (I will also assume when you said "small black hole" you really meant one that is small enough to be contained on the surface of a planet and that the 4' size was not what you really meant.)



To start with, you can't destroy black holes. You can -- with difficulty -- move them. You can feed them and make them more massive. You can charge them and change their spin. But that's about it. So pretty much all you can do is try to get it up out of the planet, and that requires as much thrust as it would to launch the same mass of ordinary material into orbit and a way to apply that thrust to the BH.



(It's true that BHs probably evaporate due to Hawking radiation (though it's never been observed), but if HR is real, evaporation takes a very long time.)



The only way moving the BH might be doable (which wouldn't necessarily destroy the planet as a side-effect) involves charging the BH and then using electromagnetic fields to accelerate it. Pretty hard to do!



That brings up the question of the BH's mass. If it was temporarily contained, then it must have had a large charge and been supported by EM fields. It's mass is its mass, and if it was a small BH -- say the same mass as a mountain -- the containment facility would need to be able to support that mass concentrated into a really, really tiny object. The Schwarzschild radius of a BH of 100,000 tons (which would be very difficult to support with any imaginable technology) would be about 10-19 meters which is roughly 1 millionth the diameter of a proton.



Something that small would find it very difficult to swallow anything, so while it would orbit inside the planet, it would grow only very, very slowly. It would probably come to a halt (due to dynamical friction) at the planet's core and just sit there growing very, very, very slowly. It would probably have no effect whatsoever on a thousand year timescale.



The bigger the BH, the more of an impact it would have, but the less likely it would be that it could ever have been contained at the planet's surface.






share|improve this answer











$endgroup$








  • 1




    $begingroup$
    I seriously doubt that it's possible to impart a meaningful charge on a black hole: Any charge you add to it will be carried away by the hawking radiation rather quickly. Yes, it may absorb many billions of electrons, but the OP's black hole is many times heavier than earth. I guess, by the time you have it charged enough to be able to move earth relative to it, it will be spitting out its charge much faster than you can add charge to it.
    $endgroup$
    – cmaster
    Jul 14 at 17:55










  • $begingroup$
    @cmaster - There's an equation for the rate a charged black holes loses charge via Hawking radiation in this paper on sci-hub.tw, see equation (7) on p. 1144 (there's also a simpler approximate equation (9) but it only works when the black hole mass is significantly larger than 1.7 * 10^5 times the sun's mass, so it won't work for small black holes). Someone could use it to figure out the rate you'd need to fire charge in so it'd be greater than the rate of charge loss.
    $endgroup$
    – Hypnosifl
    Jul 15 at 17:49











  • $begingroup$
    Unfortunately, I don't speak Russian, so that paper is out-of-reach for me. Nevertheless, for black holes of stellar weight and more, charging them is no problem (ignoring the amount of charge you need to allow electrostatic manipulation) as they have negligible hawking radiation. It's the small, hot black holes that you might stand a chance to move if they were charged, which you cannot charge because they radiate their charge away.
    $endgroup$
    – cmaster
    Jul 15 at 18:34










  • $begingroup$
    @cmaster - the paper is not in russian, the sci-hub site will usually have a captcha test where the explanatory text is in russian but you just have to type the word you see in the graphic and then hit the big button. Technically it's a pirate site but it's widely used by scientists. Anyway, if you haven't done a calculation with an equation, how do you know the BH will radiate its charge away too quickly to do useful things like using its charge to transport it a distance similar to the radius of the Earth?
    $endgroup$
    – Hypnosifl
    Jul 15 at 21:00










  • $begingroup$
    @Hypnosifl Ah, well the captcha seems to have got stuck in my uMatrix - I only saw some Russian text... Anyway, have you ever tried levitating something by charging it? Unless the thing you are levitating is very light-weight, there's no chance to be successful. Not because the electric force is too weak, but because it's so strong and there are strongly charged particles everywhere which move to neutralize it. Ok, you can put your black hole into vacuum, but once the Hawking radiation becomes hot enough to include electrons/positrons, any charge will skew the emission rates, and soon be gone.
    $endgroup$
    – cmaster
    Jul 15 at 21:34













4












4








4





$begingroup$

I will assume you want something vaguely scientifically plausible. (I will also assume when you said "small black hole" you really meant one that is small enough to be contained on the surface of a planet and that the 4' size was not what you really meant.)



To start with, you can't destroy black holes. You can -- with difficulty -- move them. You can feed them and make them more massive. You can charge them and change their spin. But that's about it. So pretty much all you can do is try to get it up out of the planet, and that requires as much thrust as it would to launch the same mass of ordinary material into orbit and a way to apply that thrust to the BH.



(It's true that BHs probably evaporate due to Hawking radiation (though it's never been observed), but if HR is real, evaporation takes a very long time.)



The only way moving the BH might be doable (which wouldn't necessarily destroy the planet as a side-effect) involves charging the BH and then using electromagnetic fields to accelerate it. Pretty hard to do!



That brings up the question of the BH's mass. If it was temporarily contained, then it must have had a large charge and been supported by EM fields. It's mass is its mass, and if it was a small BH -- say the same mass as a mountain -- the containment facility would need to be able to support that mass concentrated into a really, really tiny object. The Schwarzschild radius of a BH of 100,000 tons (which would be very difficult to support with any imaginable technology) would be about 10-19 meters which is roughly 1 millionth the diameter of a proton.



Something that small would find it very difficult to swallow anything, so while it would orbit inside the planet, it would grow only very, very slowly. It would probably come to a halt (due to dynamical friction) at the planet's core and just sit there growing very, very, very slowly. It would probably have no effect whatsoever on a thousand year timescale.



The bigger the BH, the more of an impact it would have, but the less likely it would be that it could ever have been contained at the planet's surface.






share|improve this answer











$endgroup$



I will assume you want something vaguely scientifically plausible. (I will also assume when you said "small black hole" you really meant one that is small enough to be contained on the surface of a planet and that the 4' size was not what you really meant.)



To start with, you can't destroy black holes. You can -- with difficulty -- move them. You can feed them and make them more massive. You can charge them and change their spin. But that's about it. So pretty much all you can do is try to get it up out of the planet, and that requires as much thrust as it would to launch the same mass of ordinary material into orbit and a way to apply that thrust to the BH.



(It's true that BHs probably evaporate due to Hawking radiation (though it's never been observed), but if HR is real, evaporation takes a very long time.)



The only way moving the BH might be doable (which wouldn't necessarily destroy the planet as a side-effect) involves charging the BH and then using electromagnetic fields to accelerate it. Pretty hard to do!



That brings up the question of the BH's mass. If it was temporarily contained, then it must have had a large charge and been supported by EM fields. It's mass is its mass, and if it was a small BH -- say the same mass as a mountain -- the containment facility would need to be able to support that mass concentrated into a really, really tiny object. The Schwarzschild radius of a BH of 100,000 tons (which would be very difficult to support with any imaginable technology) would be about 10-19 meters which is roughly 1 millionth the diameter of a proton.



Something that small would find it very difficult to swallow anything, so while it would orbit inside the planet, it would grow only very, very slowly. It would probably come to a halt (due to dynamical friction) at the planet's core and just sit there growing very, very, very slowly. It would probably have no effect whatsoever on a thousand year timescale.



The bigger the BH, the more of an impact it would have, but the less likely it would be that it could ever have been contained at the planet's surface.







share|improve this answer














share|improve this answer



share|improve this answer








edited Jul 14 at 1:23

























answered Jul 14 at 0:55









Mark OlsonMark Olson

12.2k1 gold badge29 silver badges49 bronze badges




12.2k1 gold badge29 silver badges49 bronze badges







  • 1




    $begingroup$
    I seriously doubt that it's possible to impart a meaningful charge on a black hole: Any charge you add to it will be carried away by the hawking radiation rather quickly. Yes, it may absorb many billions of electrons, but the OP's black hole is many times heavier than earth. I guess, by the time you have it charged enough to be able to move earth relative to it, it will be spitting out its charge much faster than you can add charge to it.
    $endgroup$
    – cmaster
    Jul 14 at 17:55










  • $begingroup$
    @cmaster - There's an equation for the rate a charged black holes loses charge via Hawking radiation in this paper on sci-hub.tw, see equation (7) on p. 1144 (there's also a simpler approximate equation (9) but it only works when the black hole mass is significantly larger than 1.7 * 10^5 times the sun's mass, so it won't work for small black holes). Someone could use it to figure out the rate you'd need to fire charge in so it'd be greater than the rate of charge loss.
    $endgroup$
    – Hypnosifl
    Jul 15 at 17:49











  • $begingroup$
    Unfortunately, I don't speak Russian, so that paper is out-of-reach for me. Nevertheless, for black holes of stellar weight and more, charging them is no problem (ignoring the amount of charge you need to allow electrostatic manipulation) as they have negligible hawking radiation. It's the small, hot black holes that you might stand a chance to move if they were charged, which you cannot charge because they radiate their charge away.
    $endgroup$
    – cmaster
    Jul 15 at 18:34










  • $begingroup$
    @cmaster - the paper is not in russian, the sci-hub site will usually have a captcha test where the explanatory text is in russian but you just have to type the word you see in the graphic and then hit the big button. Technically it's a pirate site but it's widely used by scientists. Anyway, if you haven't done a calculation with an equation, how do you know the BH will radiate its charge away too quickly to do useful things like using its charge to transport it a distance similar to the radius of the Earth?
    $endgroup$
    – Hypnosifl
    Jul 15 at 21:00










  • $begingroup$
    @Hypnosifl Ah, well the captcha seems to have got stuck in my uMatrix - I only saw some Russian text... Anyway, have you ever tried levitating something by charging it? Unless the thing you are levitating is very light-weight, there's no chance to be successful. Not because the electric force is too weak, but because it's so strong and there are strongly charged particles everywhere which move to neutralize it. Ok, you can put your black hole into vacuum, but once the Hawking radiation becomes hot enough to include electrons/positrons, any charge will skew the emission rates, and soon be gone.
    $endgroup$
    – cmaster
    Jul 15 at 21:34












  • 1




    $begingroup$
    I seriously doubt that it's possible to impart a meaningful charge on a black hole: Any charge you add to it will be carried away by the hawking radiation rather quickly. Yes, it may absorb many billions of electrons, but the OP's black hole is many times heavier than earth. I guess, by the time you have it charged enough to be able to move earth relative to it, it will be spitting out its charge much faster than you can add charge to it.
    $endgroup$
    – cmaster
    Jul 14 at 17:55










  • $begingroup$
    @cmaster - There's an equation for the rate a charged black holes loses charge via Hawking radiation in this paper on sci-hub.tw, see equation (7) on p. 1144 (there's also a simpler approximate equation (9) but it only works when the black hole mass is significantly larger than 1.7 * 10^5 times the sun's mass, so it won't work for small black holes). Someone could use it to figure out the rate you'd need to fire charge in so it'd be greater than the rate of charge loss.
    $endgroup$
    – Hypnosifl
    Jul 15 at 17:49











  • $begingroup$
    Unfortunately, I don't speak Russian, so that paper is out-of-reach for me. Nevertheless, for black holes of stellar weight and more, charging them is no problem (ignoring the amount of charge you need to allow electrostatic manipulation) as they have negligible hawking radiation. It's the small, hot black holes that you might stand a chance to move if they were charged, which you cannot charge because they radiate their charge away.
    $endgroup$
    – cmaster
    Jul 15 at 18:34










  • $begingroup$
    @cmaster - the paper is not in russian, the sci-hub site will usually have a captcha test where the explanatory text is in russian but you just have to type the word you see in the graphic and then hit the big button. Technically it's a pirate site but it's widely used by scientists. Anyway, if you haven't done a calculation with an equation, how do you know the BH will radiate its charge away too quickly to do useful things like using its charge to transport it a distance similar to the radius of the Earth?
    $endgroup$
    – Hypnosifl
    Jul 15 at 21:00










  • $begingroup$
    @Hypnosifl Ah, well the captcha seems to have got stuck in my uMatrix - I only saw some Russian text... Anyway, have you ever tried levitating something by charging it? Unless the thing you are levitating is very light-weight, there's no chance to be successful. Not because the electric force is too weak, but because it's so strong and there are strongly charged particles everywhere which move to neutralize it. Ok, you can put your black hole into vacuum, but once the Hawking radiation becomes hot enough to include electrons/positrons, any charge will skew the emission rates, and soon be gone.
    $endgroup$
    – cmaster
    Jul 15 at 21:34







1




1




$begingroup$
I seriously doubt that it's possible to impart a meaningful charge on a black hole: Any charge you add to it will be carried away by the hawking radiation rather quickly. Yes, it may absorb many billions of electrons, but the OP's black hole is many times heavier than earth. I guess, by the time you have it charged enough to be able to move earth relative to it, it will be spitting out its charge much faster than you can add charge to it.
$endgroup$
– cmaster
Jul 14 at 17:55




$begingroup$
I seriously doubt that it's possible to impart a meaningful charge on a black hole: Any charge you add to it will be carried away by the hawking radiation rather quickly. Yes, it may absorb many billions of electrons, but the OP's black hole is many times heavier than earth. I guess, by the time you have it charged enough to be able to move earth relative to it, it will be spitting out its charge much faster than you can add charge to it.
$endgroup$
– cmaster
Jul 14 at 17:55












$begingroup$
@cmaster - There's an equation for the rate a charged black holes loses charge via Hawking radiation in this paper on sci-hub.tw, see equation (7) on p. 1144 (there's also a simpler approximate equation (9) but it only works when the black hole mass is significantly larger than 1.7 * 10^5 times the sun's mass, so it won't work for small black holes). Someone could use it to figure out the rate you'd need to fire charge in so it'd be greater than the rate of charge loss.
$endgroup$
– Hypnosifl
Jul 15 at 17:49





$begingroup$
@cmaster - There's an equation for the rate a charged black holes loses charge via Hawking radiation in this paper on sci-hub.tw, see equation (7) on p. 1144 (there's also a simpler approximate equation (9) but it only works when the black hole mass is significantly larger than 1.7 * 10^5 times the sun's mass, so it won't work for small black holes). Someone could use it to figure out the rate you'd need to fire charge in so it'd be greater than the rate of charge loss.
$endgroup$
– Hypnosifl
Jul 15 at 17:49













$begingroup$
Unfortunately, I don't speak Russian, so that paper is out-of-reach for me. Nevertheless, for black holes of stellar weight and more, charging them is no problem (ignoring the amount of charge you need to allow electrostatic manipulation) as they have negligible hawking radiation. It's the small, hot black holes that you might stand a chance to move if they were charged, which you cannot charge because they radiate their charge away.
$endgroup$
– cmaster
Jul 15 at 18:34




$begingroup$
Unfortunately, I don't speak Russian, so that paper is out-of-reach for me. Nevertheless, for black holes of stellar weight and more, charging them is no problem (ignoring the amount of charge you need to allow electrostatic manipulation) as they have negligible hawking radiation. It's the small, hot black holes that you might stand a chance to move if they were charged, which you cannot charge because they radiate their charge away.
$endgroup$
– cmaster
Jul 15 at 18:34












$begingroup$
@cmaster - the paper is not in russian, the sci-hub site will usually have a captcha test where the explanatory text is in russian but you just have to type the word you see in the graphic and then hit the big button. Technically it's a pirate site but it's widely used by scientists. Anyway, if you haven't done a calculation with an equation, how do you know the BH will radiate its charge away too quickly to do useful things like using its charge to transport it a distance similar to the radius of the Earth?
$endgroup$
– Hypnosifl
Jul 15 at 21:00




$begingroup$
@cmaster - the paper is not in russian, the sci-hub site will usually have a captcha test where the explanatory text is in russian but you just have to type the word you see in the graphic and then hit the big button. Technically it's a pirate site but it's widely used by scientists. Anyway, if you haven't done a calculation with an equation, how do you know the BH will radiate its charge away too quickly to do useful things like using its charge to transport it a distance similar to the radius of the Earth?
$endgroup$
– Hypnosifl
Jul 15 at 21:00












$begingroup$
@Hypnosifl Ah, well the captcha seems to have got stuck in my uMatrix - I only saw some Russian text... Anyway, have you ever tried levitating something by charging it? Unless the thing you are levitating is very light-weight, there's no chance to be successful. Not because the electric force is too weak, but because it's so strong and there are strongly charged particles everywhere which move to neutralize it. Ok, you can put your black hole into vacuum, but once the Hawking radiation becomes hot enough to include electrons/positrons, any charge will skew the emission rates, and soon be gone.
$endgroup$
– cmaster
Jul 15 at 21:34




$begingroup$
@Hypnosifl Ah, well the captcha seems to have got stuck in my uMatrix - I only saw some Russian text... Anyway, have you ever tried levitating something by charging it? Unless the thing you are levitating is very light-weight, there's no chance to be successful. Not because the electric force is too weak, but because it's so strong and there are strongly charged particles everywhere which move to neutralize it. Ok, you can put your black hole into vacuum, but once the Hawking radiation becomes hot enough to include electrons/positrons, any charge will skew the emission rates, and soon be gone.
$endgroup$
– cmaster
Jul 15 at 21:34











4












$begingroup$

For something that sounds plausible, but totally isn't



Fire charged particles at relativistic speeds on tangents to the event horizon. And just before they cross the event horizon, slow them down to below relativistic speeds with bursts from powerful magnetic and electrical fields.



As they near the black hole, their induced mass creates gravitation fields to compete with the internal mass of the black hole. By firing multiple particles simultaneously at antipodal targets tangential to the event horizon, the black hole can't move so its internal mass it stretched between the equal gravitational forces pulling it apart.



By this means, tiny bits of matter can be teased away from the grip of the black hole.



Keep it up until it is too small to hurt the planet.



Scientists save the world, again. Your welcome.






share|improve this answer











$endgroup$








  • 3




    $begingroup$
    Sorry, but this won't work. The event horizon is really a point of no return, with the only exception of hawking radiation. Also, there's no such thing as "its internal mass it [sic] stretched between the equal gravitational forces". A black hole really behaves like a single dot of mass from any outside point of view. It cannot be deformed. You can deform the event horizon, as happens during black hole mergers, but that is purely an effect of spacetime around the black hole. The black hole itself keeps behaving as a single dot of mass.
    $endgroup$
    – cmaster
    Jul 14 at 18:03






  • 3




    $begingroup$
    @cmaster, please note the science-fiction tag and the opening clause of my answer -- its only meant to sound good. Of course, it won't work.
    $endgroup$
    – EDL
    Jul 14 at 18:26






  • 1




    $begingroup$
    Ok, I may not be the best audience for soft scifi as I've got quite a bit of physics training. And indeed, you say it's not plausible. Unfortunately, my downvote is already locked, so I can't remove it unless you edit your answer a bit. Nevertheless, the "sounds plausible" definitely does not extend to physics students.
    $endgroup$
    – cmaster
    Jul 14 at 18:41










  • $begingroup$
    @cmaster Could we reverse its polarity?
    $endgroup$
    – Sentry
    Jul 15 at 12:16










  • $begingroup$
    @Sentry Sorry, didn't understand your comment. Reverse the polarity of what?
    $endgroup$
    – cmaster
    Jul 15 at 16:45















4












$begingroup$

For something that sounds plausible, but totally isn't



Fire charged particles at relativistic speeds on tangents to the event horizon. And just before they cross the event horizon, slow them down to below relativistic speeds with bursts from powerful magnetic and electrical fields.



As they near the black hole, their induced mass creates gravitation fields to compete with the internal mass of the black hole. By firing multiple particles simultaneously at antipodal targets tangential to the event horizon, the black hole can't move so its internal mass it stretched between the equal gravitational forces pulling it apart.



By this means, tiny bits of matter can be teased away from the grip of the black hole.



Keep it up until it is too small to hurt the planet.



Scientists save the world, again. Your welcome.






share|improve this answer











$endgroup$








  • 3




    $begingroup$
    Sorry, but this won't work. The event horizon is really a point of no return, with the only exception of hawking radiation. Also, there's no such thing as "its internal mass it [sic] stretched between the equal gravitational forces". A black hole really behaves like a single dot of mass from any outside point of view. It cannot be deformed. You can deform the event horizon, as happens during black hole mergers, but that is purely an effect of spacetime around the black hole. The black hole itself keeps behaving as a single dot of mass.
    $endgroup$
    – cmaster
    Jul 14 at 18:03






  • 3




    $begingroup$
    @cmaster, please note the science-fiction tag and the opening clause of my answer -- its only meant to sound good. Of course, it won't work.
    $endgroup$
    – EDL
    Jul 14 at 18:26






  • 1




    $begingroup$
    Ok, I may not be the best audience for soft scifi as I've got quite a bit of physics training. And indeed, you say it's not plausible. Unfortunately, my downvote is already locked, so I can't remove it unless you edit your answer a bit. Nevertheless, the "sounds plausible" definitely does not extend to physics students.
    $endgroup$
    – cmaster
    Jul 14 at 18:41










  • $begingroup$
    @cmaster Could we reverse its polarity?
    $endgroup$
    – Sentry
    Jul 15 at 12:16










  • $begingroup$
    @Sentry Sorry, didn't understand your comment. Reverse the polarity of what?
    $endgroup$
    – cmaster
    Jul 15 at 16:45













4












4








4





$begingroup$

For something that sounds plausible, but totally isn't



Fire charged particles at relativistic speeds on tangents to the event horizon. And just before they cross the event horizon, slow them down to below relativistic speeds with bursts from powerful magnetic and electrical fields.



As they near the black hole, their induced mass creates gravitation fields to compete with the internal mass of the black hole. By firing multiple particles simultaneously at antipodal targets tangential to the event horizon, the black hole can't move so its internal mass it stretched between the equal gravitational forces pulling it apart.



By this means, tiny bits of matter can be teased away from the grip of the black hole.



Keep it up until it is too small to hurt the planet.



Scientists save the world, again. Your welcome.






share|improve this answer











$endgroup$



For something that sounds plausible, but totally isn't



Fire charged particles at relativistic speeds on tangents to the event horizon. And just before they cross the event horizon, slow them down to below relativistic speeds with bursts from powerful magnetic and electrical fields.



As they near the black hole, their induced mass creates gravitation fields to compete with the internal mass of the black hole. By firing multiple particles simultaneously at antipodal targets tangential to the event horizon, the black hole can't move so its internal mass it stretched between the equal gravitational forces pulling it apart.



By this means, tiny bits of matter can be teased away from the grip of the black hole.



Keep it up until it is too small to hurt the planet.



Scientists save the world, again. Your welcome.







share|improve this answer














share|improve this answer



share|improve this answer








edited Jul 15 at 1:02

























answered Jul 14 at 5:03









EDLEDL

3,8414 silver badges23 bronze badges




3,8414 silver badges23 bronze badges







  • 3




    $begingroup$
    Sorry, but this won't work. The event horizon is really a point of no return, with the only exception of hawking radiation. Also, there's no such thing as "its internal mass it [sic] stretched between the equal gravitational forces". A black hole really behaves like a single dot of mass from any outside point of view. It cannot be deformed. You can deform the event horizon, as happens during black hole mergers, but that is purely an effect of spacetime around the black hole. The black hole itself keeps behaving as a single dot of mass.
    $endgroup$
    – cmaster
    Jul 14 at 18:03






  • 3




    $begingroup$
    @cmaster, please note the science-fiction tag and the opening clause of my answer -- its only meant to sound good. Of course, it won't work.
    $endgroup$
    – EDL
    Jul 14 at 18:26






  • 1




    $begingroup$
    Ok, I may not be the best audience for soft scifi as I've got quite a bit of physics training. And indeed, you say it's not plausible. Unfortunately, my downvote is already locked, so I can't remove it unless you edit your answer a bit. Nevertheless, the "sounds plausible" definitely does not extend to physics students.
    $endgroup$
    – cmaster
    Jul 14 at 18:41










  • $begingroup$
    @cmaster Could we reverse its polarity?
    $endgroup$
    – Sentry
    Jul 15 at 12:16










  • $begingroup$
    @Sentry Sorry, didn't understand your comment. Reverse the polarity of what?
    $endgroup$
    – cmaster
    Jul 15 at 16:45












  • 3




    $begingroup$
    Sorry, but this won't work. The event horizon is really a point of no return, with the only exception of hawking radiation. Also, there's no such thing as "its internal mass it [sic] stretched between the equal gravitational forces". A black hole really behaves like a single dot of mass from any outside point of view. It cannot be deformed. You can deform the event horizon, as happens during black hole mergers, but that is purely an effect of spacetime around the black hole. The black hole itself keeps behaving as a single dot of mass.
    $endgroup$
    – cmaster
    Jul 14 at 18:03






  • 3




    $begingroup$
    @cmaster, please note the science-fiction tag and the opening clause of my answer -- its only meant to sound good. Of course, it won't work.
    $endgroup$
    – EDL
    Jul 14 at 18:26






  • 1




    $begingroup$
    Ok, I may not be the best audience for soft scifi as I've got quite a bit of physics training. And indeed, you say it's not plausible. Unfortunately, my downvote is already locked, so I can't remove it unless you edit your answer a bit. Nevertheless, the "sounds plausible" definitely does not extend to physics students.
    $endgroup$
    – cmaster
    Jul 14 at 18:41










  • $begingroup$
    @cmaster Could we reverse its polarity?
    $endgroup$
    – Sentry
    Jul 15 at 12:16










  • $begingroup$
    @Sentry Sorry, didn't understand your comment. Reverse the polarity of what?
    $endgroup$
    – cmaster
    Jul 15 at 16:45







3




3




$begingroup$
Sorry, but this won't work. The event horizon is really a point of no return, with the only exception of hawking radiation. Also, there's no such thing as "its internal mass it [sic] stretched between the equal gravitational forces". A black hole really behaves like a single dot of mass from any outside point of view. It cannot be deformed. You can deform the event horizon, as happens during black hole mergers, but that is purely an effect of spacetime around the black hole. The black hole itself keeps behaving as a single dot of mass.
$endgroup$
– cmaster
Jul 14 at 18:03




$begingroup$
Sorry, but this won't work. The event horizon is really a point of no return, with the only exception of hawking radiation. Also, there's no such thing as "its internal mass it [sic] stretched between the equal gravitational forces". A black hole really behaves like a single dot of mass from any outside point of view. It cannot be deformed. You can deform the event horizon, as happens during black hole mergers, but that is purely an effect of spacetime around the black hole. The black hole itself keeps behaving as a single dot of mass.
$endgroup$
– cmaster
Jul 14 at 18:03




3




3




$begingroup$
@cmaster, please note the science-fiction tag and the opening clause of my answer -- its only meant to sound good. Of course, it won't work.
$endgroup$
– EDL
Jul 14 at 18:26




$begingroup$
@cmaster, please note the science-fiction tag and the opening clause of my answer -- its only meant to sound good. Of course, it won't work.
$endgroup$
– EDL
Jul 14 at 18:26




1




1




$begingroup$
Ok, I may not be the best audience for soft scifi as I've got quite a bit of physics training. And indeed, you say it's not plausible. Unfortunately, my downvote is already locked, so I can't remove it unless you edit your answer a bit. Nevertheless, the "sounds plausible" definitely does not extend to physics students.
$endgroup$
– cmaster
Jul 14 at 18:41




$begingroup$
Ok, I may not be the best audience for soft scifi as I've got quite a bit of physics training. And indeed, you say it's not plausible. Unfortunately, my downvote is already locked, so I can't remove it unless you edit your answer a bit. Nevertheless, the "sounds plausible" definitely does not extend to physics students.
$endgroup$
– cmaster
Jul 14 at 18:41












$begingroup$
@cmaster Could we reverse its polarity?
$endgroup$
– Sentry
Jul 15 at 12:16




$begingroup$
@cmaster Could we reverse its polarity?
$endgroup$
– Sentry
Jul 15 at 12:16












$begingroup$
@Sentry Sorry, didn't understand your comment. Reverse the polarity of what?
$endgroup$
– cmaster
Jul 15 at 16:45




$begingroup$
@Sentry Sorry, didn't understand your comment. Reverse the polarity of what?
$endgroup$
– cmaster
Jul 15 at 16:45











3












$begingroup$

A black hole with an event horizon circumference of ~13 feet (circumference because diameter is meaningless in a region of space so deeply curved) would mass far more than the Earth. The surface gravity of an Earth-sized planet with such a body at its core would be well beyond survivability of any land animal less tough than a small spider or insect (though aquatic creatures would be okay until they tried to dive too deep -- say, more than about ten meters).



So, don't worry about destroying that four foot black hole. Everyone will be dead before it's known why the Earth is being swallowed -- and it'll never get near the center of the planet; it'll tear the whole planet up before passing without a noticeable change in trajectory.






share|improve this answer











$endgroup$








  • 2




    $begingroup$
    diameter/radius is not meaningless, you just have to select a coordinate system, usually using the radial coordinate of Schwarzschild coordinates...I explained the physical meaning of the Schwarzschild radical coordinate here, it's defined by measuring the area and then using the standard relation Area = 4 * pi * radius^2
    $endgroup$
    – Hypnosifl
    Jul 14 at 5:02















3












$begingroup$

A black hole with an event horizon circumference of ~13 feet (circumference because diameter is meaningless in a region of space so deeply curved) would mass far more than the Earth. The surface gravity of an Earth-sized planet with such a body at its core would be well beyond survivability of any land animal less tough than a small spider or insect (though aquatic creatures would be okay until they tried to dive too deep -- say, more than about ten meters).



So, don't worry about destroying that four foot black hole. Everyone will be dead before it's known why the Earth is being swallowed -- and it'll never get near the center of the planet; it'll tear the whole planet up before passing without a noticeable change in trajectory.






share|improve this answer











$endgroup$








  • 2




    $begingroup$
    diameter/radius is not meaningless, you just have to select a coordinate system, usually using the radial coordinate of Schwarzschild coordinates...I explained the physical meaning of the Schwarzschild radical coordinate here, it's defined by measuring the area and then using the standard relation Area = 4 * pi * radius^2
    $endgroup$
    – Hypnosifl
    Jul 14 at 5:02













3












3








3





$begingroup$

A black hole with an event horizon circumference of ~13 feet (circumference because diameter is meaningless in a region of space so deeply curved) would mass far more than the Earth. The surface gravity of an Earth-sized planet with such a body at its core would be well beyond survivability of any land animal less tough than a small spider or insect (though aquatic creatures would be okay until they tried to dive too deep -- say, more than about ten meters).



So, don't worry about destroying that four foot black hole. Everyone will be dead before it's known why the Earth is being swallowed -- and it'll never get near the center of the planet; it'll tear the whole planet up before passing without a noticeable change in trajectory.






share|improve this answer











$endgroup$



A black hole with an event horizon circumference of ~13 feet (circumference because diameter is meaningless in a region of space so deeply curved) would mass far more than the Earth. The surface gravity of an Earth-sized planet with such a body at its core would be well beyond survivability of any land animal less tough than a small spider or insect (though aquatic creatures would be okay until they tried to dive too deep -- say, more than about ten meters).



So, don't worry about destroying that four foot black hole. Everyone will be dead before it's known why the Earth is being swallowed -- and it'll never get near the center of the planet; it'll tear the whole planet up before passing without a noticeable change in trajectory.







share|improve this answer














share|improve this answer



share|improve this answer








edited Jul 14 at 0:46

























answered Jul 14 at 0:39









Zeiss IkonZeiss Ikon

7,59413 silver badges36 bronze badges




7,59413 silver badges36 bronze badges







  • 2




    $begingroup$
    diameter/radius is not meaningless, you just have to select a coordinate system, usually using the radial coordinate of Schwarzschild coordinates...I explained the physical meaning of the Schwarzschild radical coordinate here, it's defined by measuring the area and then using the standard relation Area = 4 * pi * radius^2
    $endgroup$
    – Hypnosifl
    Jul 14 at 5:02












  • 2




    $begingroup$
    diameter/radius is not meaningless, you just have to select a coordinate system, usually using the radial coordinate of Schwarzschild coordinates...I explained the physical meaning of the Schwarzschild radical coordinate here, it's defined by measuring the area and then using the standard relation Area = 4 * pi * radius^2
    $endgroup$
    – Hypnosifl
    Jul 14 at 5:02







2




2




$begingroup$
diameter/radius is not meaningless, you just have to select a coordinate system, usually using the radial coordinate of Schwarzschild coordinates...I explained the physical meaning of the Schwarzschild radical coordinate here, it's defined by measuring the area and then using the standard relation Area = 4 * pi * radius^2
$endgroup$
– Hypnosifl
Jul 14 at 5:02




$begingroup$
diameter/radius is not meaningless, you just have to select a coordinate system, usually using the radial coordinate of Schwarzschild coordinates...I explained the physical meaning of the Schwarzschild radical coordinate here, it's defined by measuring the area and then using the standard relation Area = 4 * pi * radius^2
$endgroup$
– Hypnosifl
Jul 14 at 5:02











1












$begingroup$

Spin it up until its angular momentum becomes sufficiently large that it becomes a naked, toroidal singularity, at which point the laws of physics metaphorically throw up their hands and shrug in confusion.






share|improve this answer









$endgroup$








  • 3




    $begingroup$
    Which will instantly annihilate earth in a giant burst of gamma radiation...
    $endgroup$
    – cmaster
    Jul 14 at 18:04















1












$begingroup$

Spin it up until its angular momentum becomes sufficiently large that it becomes a naked, toroidal singularity, at which point the laws of physics metaphorically throw up their hands and shrug in confusion.






share|improve this answer









$endgroup$








  • 3




    $begingroup$
    Which will instantly annihilate earth in a giant burst of gamma radiation...
    $endgroup$
    – cmaster
    Jul 14 at 18:04













1












1








1





$begingroup$

Spin it up until its angular momentum becomes sufficiently large that it becomes a naked, toroidal singularity, at which point the laws of physics metaphorically throw up their hands and shrug in confusion.






share|improve this answer









$endgroup$



Spin it up until its angular momentum becomes sufficiently large that it becomes a naked, toroidal singularity, at which point the laws of physics metaphorically throw up their hands and shrug in confusion.







share|improve this answer












share|improve this answer



share|improve this answer










answered Jul 14 at 10:38









nick012000nick012000

1,3103 silver badges12 bronze badges




1,3103 silver badges12 bronze badges







  • 3




    $begingroup$
    Which will instantly annihilate earth in a giant burst of gamma radiation...
    $endgroup$
    – cmaster
    Jul 14 at 18:04












  • 3




    $begingroup$
    Which will instantly annihilate earth in a giant burst of gamma radiation...
    $endgroup$
    – cmaster
    Jul 14 at 18:04







3




3




$begingroup$
Which will instantly annihilate earth in a giant burst of gamma radiation...
$endgroup$
– cmaster
Jul 14 at 18:04




$begingroup$
Which will instantly annihilate earth in a giant burst of gamma radiation...
$endgroup$
– cmaster
Jul 14 at 18:04



Popular posts from this blog

Get product attribute by attribute group code in magento 2get product attribute by product attribute group in magento 2Magento 2 Log Bundle Product Data in List Page?How to get all product attribute of a attribute group of Default attribute set?Magento 2.1 Create a filter in the product grid by new attributeMagento 2 : Get Product Attribute values By GroupMagento 2 How to get all existing values for one attributeMagento 2 get custom attribute of a single product inside a pluginMagento 2.3 How to get all the Multi Source Inventory (MSI) locations collection in custom module?Magento2: how to develop rest API to get new productsGet product attribute by attribute group code ( [attribute_group_code] ) in magento 2

Category:9 (number) SubcategoriesMedia in category "9 (number)"Navigation menuUpload mediaGND ID: 4485639-8Library of Congress authority ID: sh85091979ReasonatorScholiaStatistics

Magento 2.3: How do i solve this, Not registered handle, on custom form?How can i rewrite TierPrice Block in Magento2magento 2 captcha not rendering if I override layout xmlmain.CRITICAL: Plugin class doesn't existMagento 2 : Problem while adding custom button order view page?Magento 2.2.5: Overriding Admin Controller sales/orderMagento 2.2.5: Add, Update and Delete existing products Custom OptionsMagento 2.3 : File Upload issue in UI Component FormMagento2 Not registered handleHow to configured Form Builder Js in my custom magento 2.3.0 module?Magento 2.3. How to create image upload field in an admin form